C2 - Problem Examples
C2 - Problem Examples
375 N
MP F cos ( T ) ( a c) F sin ( T ) ( b d)
MP
737 N m
Problem 4-102
Replace the force system by an equivalent force and couple moment at point O.
Units Used :
3
kip 10 lb
Given :
F1 430lb
F2 260lb
a 2ft
e 5ft
b 8ft
f 12
c 3ft
g 5
d a
T 60deg
Solution:
F sin ( T )
1
2
2
g
f
FRx
272.39lb
F cos ( T )
1
2
2
g f
FRy
25 lb
FR
274 lb
T1
5.24 deg
FRx F 2
FRy F 2
FR
FRx FRy
FRy
T 1 atan
FRx
Mo F 1 cos ( T ) ( a) F1 sin ( T ) ( b) F2
f
2
g f
Mo
4.61 kip ft
Problem 4-125
Replace the force and couple-moment system by an equivalent resultant force and couple
moment at point O. Express the results in Cartesian vector form.
Units Used:
kN 1000N
Given:
M = Mx i My j Mz k
F = F x i Fy j Fz k
Mx 20kN m
Fx 8kN
My 70kN m
Fy 6kN
Mz 20kN m
Fz 8kN
a 3m
b 3m
e 5m
c 4m
f 6m
d 6m
g 5m
Solution:
Fx
F Fy
Fz
FR F
Mx
M My
Mz
MR M r u F
f
r e
g
FR
8
6 kN
MR
10
18 kN m
56
Problem 4-127
Replace the force and couple-moment system by an equivalent resultant force and couple
moment at point Q. Express the results in Cartesian vector form.
Units Used:
kN 1000N
Given:
M = Mx i My j Mz k
F = Fx i Fy j Fz k
Mx 20kN m
Fx 8kN
My 70kN m
Fy 6kN
Mz 20kN m
Fz 8kN
a 3m
b 3m
e 5m
c 4m
f 6m
d 6m
g 5m
Solution:
Fx
F Fy
F
z
FR F
Mx
M My
M
z
MR M r u F
0
e
r
FR
8
6 kN
MR
10
30 kN m
20
Problem 4-161
Determine the coordinate direction angles of F, which is applied to the end A of the pipe
assembly, so that the moment of F about O is zero.
Units Used:
Given:
F 20lb
a 8in
b 6in
c 6in
d 10in
Solution:
Require Mo = 0. This happens when force F is directed either towards or away from point O.
c
r a b
d
u
r
r
0.3
0.8
0.5
D
E acos ( u)
J
D
E
J
70.8
39.8 deg
56.7
D
E acos ( u)
J
D
E
J
109.2
140.2 deg
123.3
Problem 4-162
Determine the moment of the force F about point O. The force has coordinate direction angles
D, E, J. Express the result as a Cartesian vector.
Given:
F 20lb
a 8in
D 60deg
b 6in
E 120deg
c 6in
J 45deg
d 10in
Solution:
c
r a b
d
cos ( D )
Fv F cos ( E )
cos ( J )
M r u Fv
298.0
15.1 lb in
200.0
Problem 4-166
Determine the resultant couple moment of the two couples that act on the assembly. Member
OB lies in the x-z plane.
Given :
F1 400N
F2 150N
a 500mm
b 600mm
c 400mm
T 45deg
Solution:
F1
F1v 0
0
0
F2v F2
0
b cos T
r1B
0
b sin T
0
r1A a
b cos T
c
r2
b sin T
63.64
M 169.71
Nm
263.64
Problem 4-167
Replace the force F having acting at point A by an equivalent force and couple moment at point
C.
Given:
F 50lb
a 10ft
b 20ft
c 15ft
d 10ft
e 30ft
Solution:
d
rAB c
e
Fv F
rAB
rAB
0
rCA a b
e
FR Fv
MR rCA u F v
FR
14.3
21.4 lb
42.9
MR
1928.6
428.6 lb ft
428.6
Problem 4-169
The horizontal force F acts on the handle of the wrench. Determine the moment of this force
about point O. Specify the coordinate direction angles D, E, J of the moment axis.
Given:
F 30N
a 50mm
b 200mm
c 10mm
T 45deg
Solution :
sin ( T )
Fv F cos ( T )
MO rOA u F v
D
MO
E acos
J
MO
c
rOA b
a
MO
D
E
J
1.06
1.06 N m
4.03
75.7
75.7 deg
159.6
Problem 4-137
Replace the three forces acting on the plate by a wrench. Specify the magnitude of the force
and couple moment for the wrench and the point P(x, y) where its line of action intersects the
plate.
Units Used:
kN 10 N
Given :
FA 500N
a 4m
FB 800N
b 6m
FC 300N
Solution:
FA
FR FC
FB
Guesses
x 1m
y 1m
M 100N m
Given
M
y u FR =
FR
0
FR
M
x Find ( M x y)
y
b 0 0 0
a u F a u 0
C
0 0 0 FB
3.07 kN m
1.163 m
2.061
Problem 4-138
Replace the three forces acting on the plate by a wrench. Specify the magnitude of the force and
couple moment for the wrench and the point P(y,z) where its line of action intersects the plate.
Units Used :
Given :
FA 80lb
a 12ft
FB 60lb
b 12ft
FC 40lb
Solution:
FC
FR FB
FA
Guesses
Given
y 1ft
z 1ft
M
y u FR =
FR
z
FR
M
y Find ( M y z)
z
M 1lb ft
0 FC 0 0
a u
0 a u FB
0 0 b 0
624 lb ft
0.414 ft
8.69
y
F1 = 600 N
F2 = 500 N
The forces F1, F2, and F3, all of which act on point A of the bracket, are
specied in three different ways. Determine the x and y scalar components of
each of the three forces.
35
0.1 m
x
Solution.
0.2 m
Ans.
Ans.
0.3 m
F3 = 800 N
B
0.4 m
Ans.
3
F2y 500(5) 300 N
Ans.
F1 x
Ans.
Ans.
F3 F3nAB F3
F3 y
0.2 m
4
A
F2 x
0.4 m
(c)
(b)
Helpful Hints
0.2i 0.4j
AB
800
AB
(0.2)2 (0.4)2
800 [0.447i 0.894j]
358i 716j N
Ans.
F3y 716 N
Ans.
00
F2 y
(a)
F2 = 500 N
35
A
=8
26.6
0.2
0.4
F3 x
A
F3
Note that the angle which orients F2 to the x-axis is never calculated. The cosine
and sine of the angle are available by inspection of the 3-4-5 triangle. Also note
that the x scalar component of F2 is negative by inspection.
The scalar components of F3 can be obtained by rst computing the angle
of Fig. c.
tan1
F1 = 600 N
F1 y
P = 800 lb
Combine the two forces P and T, which act on the xed structure at B, into
a single equivalent force R.
lb
60
Graphical solution.
P is constructed as shown in Fig. a. The scale used here is 1 in. 800 lb; a scale
C 60
of 1 in. 200 lb would be more suitable for regular-size paper and would give
greater accuracy. Note that the angle a must be determined prior to construction
of the parallelogram. From the given gure
tan
6 sin 60
BD
0.866
3
6 cos 60
AD
49
00
lb
R
(a)
Helpful Hints
Ans.
Geometric solution.
800 lb
40.9
Measurement of the length R and direction of the resultant force R yields the
approximate results
R 525 lb
Ans.
B
From the law of sines, we may determine the angle which orients R. Thus,
524
600
800 lb
600 lb
sin 0.750
48.6
Ans.
T
(b)
Algebraic solution.
we may write
tan1
Ry
Rx
(346)2
tan
(393)2
524 lb
393
48.6
346
Ans.
Ans.
Rx = 346 lb
Ry = 393 lb
Ans.
(c)
y
j
The 500-N force F is applied to the vertical pole as shown. (1) Write F in
terms of the unit vectors i and j and identify both its vector and scalar components. (2) Determine the scalar components of the force vector F along the
x- and y-axes. (3) Determine the scalar components of F along the x- and y-axes.
y
30
x
j
A
i
Solution.
Part (1).
F = 500 N
30
F (F cos )i (F sin )j
x
y
Ans.
Fx
The scalar components are Fx 250 N and Fy 433 N. The vector components are Fx 250i N and Fy 433j N.
Part (2). From Fig. b we may write F as F 500i N, so that the required
scalar components are
Fx 500 N
Fy 0
Fy
sin 60
500
sin 30
F
F
i
Ans.
(a)
(b)
y
30
F = 500 N
Fy 866 N
(c)
Helpful Hint
Fy 866 N
Ans.
30
20
b
F2 = 80 N
(80)2
(100)2
30
90
60
R2
Fx
60
90
Fy
Fx 1000 N
= 60
Fy
Part (3). The components of F in the x- and y-directions are nonrectangular and are obtained by completing the parallelogram as shown in Fig. c. The
magnitudes of the components may be calculated by the law of sines. Thus,
Fx
500
R 163.4 N
0N
The gure also shows the orthogonal projection Fb of R onto the b-axis. Its
length is
Fb 80 100 cos 50 144.3 N
F1
Ans.
Note that the components of a vector are in general not equal to the projections of the vector onto the same axes. If the a-axis had been perpendicular to
the b-axis, then the projections and components of R would have been equal.
10
50
80 N
F2
50
Fb
b
2m
A
Calculate the magnitude of the moment about the base point O of the 600-N
force in ve different ways.
40
600 N
4m
Solution.
(I)
40
Ans.
4m
600 N
40 d
F1 = 600 cos 40
2m
F2 = 600 sin 40
(III) By the principle of transmissibility, move the 600-N force along its
line of action to point B, which eliminates the moment of the component F2. The
moment arm of F1 becomes
O
F2
MO 460(5.68) 2610 N m
F1
A
Ans.
(IV) Moving the force to point C eliminates the moment of the component
F1. The moment arm of F2 becomes
d2 2 4 cot 40 6.77 m
d1
C
O
d2
F2
MO 386(6.77) 2610 N m
Ans.
(V) By the vector expression for a moment, and by using the coordinate
system indicated on the gure together with the procedures for evaluating cross
products, we have
shortest approach.
2610k N m
The minus sign indicates that the vector is in the negative z-direction. The magnitude of the vector expression is
MO 2610 N m
Helpful Hints
Ans.
F1
The trap door OA is raised by the cable AB, which passes over the small frictionless guide pulleys at B. The tension everywhere in the ca`le is T, and this tension applied at A causes a moment MO about the hinge at O. Plot the quantity MO/T
as a function of the door elevation angle over the range 0 90 and note minimum and maximum values. What is the physical signicance of this ratio?
0.5 m
O
Solution.
rAB
T
rOB
0.3 m
A
0.4 m
rOA
So
rAB rOB rOA 0.4j (0.5)(cos i sin j)
and
rAB (0.5 cos )2 (0.4 0.5 sin )2
0.5
0.4
MO
,
T
0.4 sin
0.4 sin
0.2T cos
0.41 0.4 sin
sition vector r runs from the moment center to any point on the line
of action of F. Here, rOB is more convenient than rOA.
0.2T cos
0.41 0.4 sin
T
0.41 0.4 sin
Recall that any vector may be written as a magnitude times an aiming unit vector.
The magnitude of MO is
MO
0 10 20 30 40 50 60 70 80 90
, deg
0.2
0.1
T TnAB T
MO 0.4j T
0.3
m
Ans.
which is plotted in the accompanying graph. The expression MO/T is the moment
arm d (in meters) which runs from O to the line of action of T. It has a maximum
value of 0.4 m at 53.1 (at which point T is horizontal) and a minimum value of
0 at 90 (at which point T is vertical). The expression is valid even if T varies.
This sample problem treats moments in two-dimensional force systems, and
it also points out the advantages of carrying out a solution for an arbitrary position, so that behavior over a range of positions can be examined.
40
P
100
Solution.
[M Fd]
M 100(0.1) 10 N m
100
M 400(0.040) cos
100 N
100 N
Dimensions in millimeters
10 (400)(0.040) cos
10
cos1 16 51.3
P = 400 N
Ans.
Helpful Hint
40 mm
Since the two equal couples are parallel free vectors, the only dimensions
which are relevant are those which give the perpendicular distances between
the forces of the couples.
P = 400 N
80 lb
Replace the horizontal 80-lb force acting on the lever by an equivalent system consisting of a force at O and a couple.
We apply two equal and opposite 80-lb forces at O and identify the
counterclockwise couple
[M Fd]
60
Solution.
Ans.
80 lb
80 lb
Thus, the original force is equivalent to the 80-lb force at O and the 624-lb-in.
couple as shown in the third of the three equivalent gures.
Helpful Hint
80 lb 80 lb
O
80 lb
624 lb-in.
Determine the resultant of the four forces and one couple which act on the
plate shown.
2m
5m
60 N
50 N
45
Solution.
2m
[Rx Fx]
2m
[Ry Fy]
140 Nm
80 N
[R Rx2 Ry2]
tan
Ry
Ans.
132.4
63.2
66.9
Ans.
tan
Rx
[MO (Fd)]
40 N
1m
y
R = 148.3 N
(a)
237 N m
= 63.2
|MO| =
237 Nm
148.3d 237
d 1.600 m
Ry b MO
and
(b)
r R MO
where r xi yj is a position vector running from point O to any point on the
line of action of R. Substituting the vector expressions for r, R, and MO and carrying out the cross product result in
(xi yj) (66.9i 132.4j) 237k
(132.4x 66.9y)k 237k
Thus, the desired line of action, Fig. c, is given by
132.4x 66.9y 237
63.2
1.600 m
A
O
(c)
132.4 x 66.9 y =
237
O
R
x
B
237
1.792 m
b
132.4
Alternatively, the y-intercept could have been obtained by noting that the moment about O would be due to Rx only.
A more formal approach in determining the nal line of action of R is to use
the vector expression
R = 148.3 N
Ans.
Hence, the resultant R may be applied at any point on the line which makes a
63.2 angle with the x-axis and is tangent at point A to a circle of 1.600-m radius
with center O, as shown in part b of the gure. We apply the equation Rd MO in
an absolute-value sense (ignoring any sign of MO) and let the physics of the situation, as depicted in Fig. a, dictate the nal placement of R. Had MO been counterclockwise, the correct line of action of R would have been the tangent at point B.
The resultant R may also be located by determining its intercept distance b
to point C on the x-axis, Fig. c. With Rx and Ry acting through point C, only Ry
exerts a moment about O so that
30
Helpful Hints
F = 100 N
z
4m
A
5m
2m
6m
Solution.
F FnOA F
3i 4j 5k
3m
6m
OA
100
OA
32 42 52
x
z
Fx 42.4 N
Part (b).
Fz
Fy 56.6 N
Fz 70.7 N
32 42
32 42 52
Fy
xy
0.707
Ans.
Fxy = 70.7 N
Ans.
Fx
nOB
6i 6j 2k
OB
nOB
Ans.
y
FOB = 84.4 N
O
x
Helpful Hints
Determine the moment of force F about point O (a) by inspection and (b) by
the formal cross-product denition MO r F.
a
F
Solution. (a) Because F is parallel to the y-axis, F has no moment about that
axis. It should be clear that the moment arm from the x-axis to the line of action
of F is c and that the moment of F about the x-axis is negative. Similarly, the
moment arm from the z-axis to the line of action of F is a and the moment of F
about the z-axis is positive. So we have
MO cFi aFk F(ci ak)
x
z
Ans.
(b) Formally,
F
r
Ans.
Helpful Hint
Again we stress that r runs from the moment center to the line of action of F.
Another permissible, but less convenient, position vector is r ai bj ck.
1.6 m
The turnbuckle is tightened until the tension in cable AB is 2.4 kN. Determine the moment about point O of the cable force acting on point A and the
magnitude of this moment.
2m
Solution.
O
y
T TnAB 2.4
0.8 1.5 2
0.8i 1.5j 2k
2
1.5 m
0.8 m
z
1.6 m
rOA
Ans.
2m
Ans.
Helpful Hint
The student should verify by inspection the signs of the moment components.
y
x
1.5 m
B
0.8 m
15 m
Solution (a).
The required moment may be obtained by nding the component along the z-axis of the moment MO of T about point O. The vector MO is
normal to the plane dened by T and point O, as shown in the accompanying gure. In the use of Eq. 2/14 to nd MO, the vector r is any vector from point O to
the line of action of T. The simplest choice is the vector from O to A, which is
written as r 15j m. The vector expression for T is
T TnAB 10
O
x
z
9m
12 m
B
Helpful Hints
T = 10 kN
150(0.566k 0.424i) kN m
The value Mz of the desired moment is the scalar component of MO in the
z-direction or Mz MO k. Therefore,
Mz 150(0.566k 0.424i) k 84.9 kN m
Ans.
The minus sign indicates that the vector Mz is in the negative z-direction. Ex-
y
A
Solution (b).
O
x
The moment arm d equals OA multiplied by the sine of the angle between Txy
and OA, or
z
B
12
9.37 m
122 152
Mo
d 15
Mz
Tz
Tx
Ans.
Ty
Txy 15 m
Solution (c). The component Txy is further resolved into its components Tx and Ty.
It is clear that Ty exerts no moment about the z-axis since it passes through it, so
that the required moment is due to Tx alone. The direction cosine of T with respect
to the x-axis is 12/92 122 152 0.566 so that Tx 10(0.566) 5.66 kN. Thus,
Mz 5.66(15) 84.9 kN m
Ans.
x
9m
12 m
B
30 N
30 N
60
Determine the magnitude and direction of the couple M which will replace
the two given couples and still produce the same external effect on the block.
Specify the two forces F and F, applied in the two faces of the block parallel to
the y-z plane, which may replace the four given forces. The 30-N forces act parallel to the y-z plane.
60
100 mm
mm
60
40
mm y
25 N
50 mm
Solution. The couple due to the 30-N forces has the magnitude M1 30(0.06)
1.80 N m. The direction of M1 is normal to the plane dened by the two forces,
and the sense, shown in the gure, is established by the right-hand convention.
The couple due to the 25-N forces has the magnitude M2 25(0.10) 2.50 N m
with the direction and sense shown in the same gure. The two couple vectors
combine to give the components
25 N
z
M2 = 2.5 Nm
M
x
Thus,
y
F
Ans.
with
tan1
1.559
1 0.974 44.3
1.600 tan
Ans.
The forces F and F lie in a plane normal to the couple M, and their moment arm as seen from the right-hand gure is 100 mm. Thus, each force has the
magnitude
2.23
F 0.10 22.3 N
[M = Fd]
Ans.
60
M1 = 1.8 Nm
z
Helpful Hint
A force of 40 lb is applied at A to the handle of the control lever which is attached to the xed shaft OB. In determining the effect of the force on the shaft
at a cross section such as that at O, we may replace the force by an equivalent
force at O and a couple. Describe this couple as a vector M.
40 lb
8
2
3
A
O
Solution.
y
x
tan1 8 32.0
40 lb
Ans.
The couple vector is perpendicular to the plane in which the force is shifted, and
its sense is that of the moment of the given force about O. The direction of M in
the y-z plane is given by
Ans.
(40 lb)
O
x
A
5
8
y
z
700 lb-in.
Determine the resultant of the force and couple system which acts on the
rectangular solid.
50 lb
80 lb
Solution.
50 lb
960
lb-in.
12
1000 lb-in.
80 lb
16
10 100 lb
100 lb
Hence, the resultant consists of a couple, which of course may be applied at any
point on the body or the body extended.
Helpful Hints
Since the force summation is zero, we conclude that the resultant, if it exists,
must be a couple.
The moments associated with the force pairs are easily obtained by using the
50 N
Determine the resultant of the system of parallel forces which act on the
plate. Solve with a vector approach.
0.5 m
z
Solution.
0.5 m
500 N
0.35 m
200 N
300 N
z
x
r R MO
x
z
350z 87.5
Hence, x 0.357 m and z 0.250 m are the coordinates through which the
line of action of R must pass. The value of y may, of course, be any value, as
permitted by the principle of transmissibility. Thus, as expected, the variable y
drops out of the above vector analysis.
From the one vector equation we may obtain the two scalar equations
and
0.35 m
MO
Helpful Hint
x
700 N
Replace the two forces and the negative wrench by a single force R applied
at A and the corresponding couple M.
500 N
25 N m
60
40
z B
Solution.
[Rx Fx]
[Ry Fy]
[Rz Fz]
80 mm
45
600 N
40 mm
120 mm
Thus,
and
100
mm
30
mm
A 50 mm
60 mm
Ans.
Helpful Hints
The moment of the 600-N force about A is written by inspection of its x- and zcomponents, which gives
wrench points in the direction opposite to that of the 500-N force, and
we must resolve it into its x-, y-, and
z-components to be added to the
other couple-vector components.
Ans.
Determine the wrench resultant of the three forces acting on the bracket.
Calculate the coordinates of the point P in the x-y plane through which the resultant force of the wrench acts. Also nd the magnitude of the couple M of the
wrench.
4
3
y
40 lb
Solution.
P
40 lb
x
20 lb
The moment of the wrench couple must equal the sum of the moments of
the given forces about point P through which R passes. The moments about P of
the three forces are
(M)Rx 20yk lb-in.
x=
3i
n.
P
5
y = 2 in.
M
Helpful Hint
is positive. If M turns out to be negative, then the direction of the couple vector is opposite to that of the
resultant force.
M
3
200 40x
2M
3
40x 20y
2M
3
x 3 in.
y 2 in.
Ans.
We see that M turned out to be negative, which means that the couple vector is
pointing in the direction opposite to R, which makes the wrench negative.